

- PowerScore Staff
- Posts: 6030
- Joined: Mar 25, 2011
- Fri Jul 09, 2021 2:32 pm
#88618
Complete Question Explanation
(The complete setup for this game can be found here: lsat/viewtopic.php?f=225&t=35163)
The correct answer choice is (A)
A Global question requires you to examine your diagram for the information needed to evaluate each answer choice. In this instance, our diagram primarily consists of the super-sequence, so use that to attack each answer choice.
Answer choice (A): This is the correct answer choice. As shown in the diagram, O must always row somewhere behind V, which rows somewhere behind S. Thus, O can never row ahead of S.
Answer choice (B): Because the only rule involving Z states that Z must row closer to the front than O, it is possible for Z to row closer to the front than M. Therefore, this answer choice could be true and it is incorrect.
Answer choice (C): The second rule states that L and V row somewhere behind S. However, this rule does not indicate that L rows ahead of V, or that V rows ahead of L. Consequently, it is possible for L to row closer to the front than V, and this answer choice is incorrect. The hypothetical produced in question #8 also helps show that this scenario could occur.
Answer choice (D): Although Z could be as far forward as seat 1, Z could also be as far back as seat 5. Hence, it is possible for S to row closer to the front than Z, and this answer choice is incorrect. As with answer choice (C), the hypothetical produced in question #8 helps show that this scenario is possible.
Answer choice (E): This answer choice can be eliminated by the similar reasoning used to eliminate answer choice (C). Just as we were able to show that L could row closer to the front than V, it is possible that V can row closer to the front than L.
(The complete setup for this game can be found here: lsat/viewtopic.php?f=225&t=35163)
The correct answer choice is (A)
A Global question requires you to examine your diagram for the information needed to evaluate each answer choice. In this instance, our diagram primarily consists of the super-sequence, so use that to attack each answer choice.
Answer choice (A): This is the correct answer choice. As shown in the diagram, O must always row somewhere behind V, which rows somewhere behind S. Thus, O can never row ahead of S.
Answer choice (B): Because the only rule involving Z states that Z must row closer to the front than O, it is possible for Z to row closer to the front than M. Therefore, this answer choice could be true and it is incorrect.
Answer choice (C): The second rule states that L and V row somewhere behind S. However, this rule does not indicate that L rows ahead of V, or that V rows ahead of L. Consequently, it is possible for L to row closer to the front than V, and this answer choice is incorrect. The hypothetical produced in question #8 also helps show that this scenario could occur.
Answer choice (D): Although Z could be as far forward as seat 1, Z could also be as far back as seat 5. Hence, it is possible for S to row closer to the front than Z, and this answer choice is incorrect. As with answer choice (C), the hypothetical produced in question #8 helps show that this scenario is possible.
Answer choice (E): This answer choice can be eliminated by the similar reasoning used to eliminate answer choice (C). Just as we were able to show that L could row closer to the front than V, it is possible that V can row closer to the front than L.
Dave Killoran
PowerScore Test Preparation
Follow me on X/Twitter at http://twitter.com/DaveKilloran
My LSAT Articles: http://blog.powerscore.com/lsat/author/dave-killoran
PowerScore Podcast: http://www.powerscore.com/lsat/podcast/
PowerScore Test Preparation
Follow me on X/Twitter at http://twitter.com/DaveKilloran
My LSAT Articles: http://blog.powerscore.com/lsat/author/dave-killoran
PowerScore Podcast: http://www.powerscore.com/lsat/podcast/